XEVIAN_ZONG
Thanks Received: 0
Vinny Gambini
Vinny Gambini
 
Posts: 10
Joined: February 28th, 2011
 
 
 

Q9 - Measurement of the motion

by XEVIAN_ZONG Mon Feb 28, 2011 11:17 pm

Hello, can you please explain to me why the answer B is the correct one? Also, I don't really get the logic relationship between the information provided in the stem of the questions. Thanks!
 
giladedelman
Thanks Received: 833
LSAT Geek
 
Posts: 619
Joined: April 04th, 2010
 
This post thanked 1 time.
 
 

Re: Q9 - Measurement of the motion

by giladedelman Wed Mar 02, 2011 8:05 pm

Thanks for posting!

Measurements of Uranus's orbit seem to show it being tugged away from the sun and the other planets by some other force -- and neither Pluto nor Neptune, the only planets on the far side of Uranus, are big enough to exert that kind of gravitational force. From these facts, the argument concludes that there must be some other planet we don't know about.

Well, okay, that could explain the measurements, but does it have to be true? No! The argument is making a big assumption: only planets can exert the kind of force that explains Uranus's motion. What if something else were responsible -- like a meteor, or debris from an alien spaceship? (Just sayin'...)

That's why (B) is correct. A belt of comets with a strong gravitational pull is exactly the kind of alternative explanation that would weaken this argument.

(A) is out of scope -- who cares when Pluto was discovered?

(C ) doesn't matter: we already know those planets aren't big enough.

(D) is tempting, but this doesn't explain why there seems to be a force pulling Uranus away.

(E) is totally out of scope.

Does that answer your question?
 
XEVIAN_ZONG
Thanks Received: 0
Vinny Gambini
Vinny Gambini
 
Posts: 10
Joined: February 28th, 2011
 
 
 

Re: Q9 - Measurement of the motion of the planet Uranus

by XEVIAN_ZONG Fri Mar 04, 2011 9:13 pm

Yes! Now it is much clearer to me. Thank you!
 
roflcoptersoisoi
Thanks Received: 0
Atticus Finch
Atticus Finch
 
Posts: 165
Joined: April 30th, 2015
 
 
 

Re: Q9 - Measurement of the motion

by roflcoptersoisoi Thu May 26, 2016 9:02 pm

giladedelman Wrote:Thanks for posting!

Measurements of Uranus's orbit seem to show it being tugged away from the sun and the other planets by some other force -- and neither Pluto nor Neptune, the only planets on the far side of Uranus, are big enough to exert that kind of gravitational force. From these facts, the argument concludes that there must be some other planet we don't know about.

Well, okay, that could explain the measurements, but does it have to be true? No! The argument is making a big assumption: only planets can exert the kind of force that explains Uranus's motion. What if something else were responsible -- like a meteor, or debris from an alien spaceship? (Just sayin'...)

That's why (B) is correct. A belt of comets with a strong gravitational pull is exactly the kind of alternative explanation that would weaken this argument.

(A) is out of scope -- who cares when Pluto was discovered?

(C ) doesn't matter: we already know those planets aren't big enough.

(D) is tempting, but this doesn't explain why there seems to be a force pulling Uranus away.

(E) is totally out of scope.

Does that answer your question?



Great explanation, although I disagree a little bit with your reasoning for eliminating (D), if the Sun exerts less force on Uranus relative to other planets then could that not also conceivably explain why Uranus may start to gravitate away from the sun and the inner planets, towards pluto and Neptune? I myself am trying to come up with a more compelling reason to eliminate (D), if anyone has one, let me know.

EDIT: Nevermind, you're exactly right, stimulus explicitly says that a force is tugging Uranus away from inner planets, it isn't just gravitating there on it's own volition, in this case (D) is irrelevant.